8
$\begingroup$

Suppose $\alpha > 1$ is irrational. Are there infinitely many primes of the form $\left\lfloor \alpha n \right\rfloor$? Is the number of $p \leq X$ of this form $\sim \alpha^{-1} X (\log{X})^{-1}$? I know this is the kind of thing the circle method was born to do, but I cannot for the life of me find a reference for this!

$\endgroup$
2
  • 5
    $\begingroup$ Please reformulate the question in such a way that you don't ask for infinitely many primes p ≤ X. $\endgroup$ May 21, 2010 at 18:36
  • $\begingroup$ @FGD: Whoops! Done. $\endgroup$ May 22, 2010 at 18:08

2 Answers 2

8
$\begingroup$

I think the uniform distribution mod1 of $\{p/\alpha\}$ is due to Vinogradov, and the asmptotic for primes in a Beatty sequence $\sim \frac{\pi(x)}{\alpha}$ is an immediate consequence. Indeed for $p$ to be equal to some $\lfloor k\alpha\rfloor$ it is equivalent to $1-\frac{1}{\alpha}<\frac{p}{\alpha}-\lfloor \frac{p}{\alpha}\rfloor<1$. So you just need the fractional part of $p/\alpha$ to be on a fixed interval of length $\alpha$ mod1.

On a related note this paper discusses the general sequence $q\lfloor \alpha n+\beta\rfloor +a$.

$\endgroup$
2
  • $\begingroup$ "...is an immediate consequence." No, it's not, because n doesn't have to be prime for $\left \lfloor \alpha n \right \rfloor$ to be prime. $\endgroup$ May 21, 2010 at 17:45
  • $\begingroup$ Sorry for stating it like that, I added a clarification. $\endgroup$ May 21, 2010 at 18:11
6
$\begingroup$

Imre Ruzsa observed that since $p/\alpha$ is equidistributed modulo $1$, we have infinitely many primes $p$ for which the fractional part of $p/\alpha$ is less than $1/\alpha$. Writing $p/\alpha = n_p - {\epsilon}_p$ with $n_p$ an integer and $0 < {\epsilon}_p < 1/\alpha$, we get $p = {\alpha}n_p - {\alpha}{\epsilon}_p$ and thus ${\lfloor}{\alpha}n_p{\rfloor}$ prime for infinitely many distinct positive integers $n_p$.

$\endgroup$
2
  • $\begingroup$ Yeah, I wasn't sure what the "local obstructions" for $\left\lfloor \alpha n \right \rfloor$ being prime would be exactly! $\endgroup$ May 21, 2010 at 16:24
  • $\begingroup$ I didn't quite remember the details correctly, but I have fixed them now. I would have to think more about the asymptotic estimate, so I have withdrawn my remarks on that. $\endgroup$
    – engelbrekt
    May 21, 2010 at 16:34

Your Answer

By clicking “Post Your Answer”, you agree to our terms of service and acknowledge you have read our privacy policy.

Not the answer you're looking for? Browse other questions tagged or ask your own question.